0 Daumen
438 Aufrufe

Aufgabe:

Es soll gezeigt werden, dass

yn \( \sqrt{a} \) ≤ xn 

für alle n ≥ 1 gilt.

Folgende Informationen sind in der Aufgabe gegeben:

yn := \( \frac{a}{x_n} \)

xn+1 := 1/2(xn+\( \frac{a}{x_n}) \)


Problem/Ansatz:

Ich hatte versucht, dieses Problem mit Hilfe einer Ungleichung zu lösen, was allerdings nicht geklappt hat.

\( \frac{a}{x_n} \) ≤ \( \sqrt{a} \) | 2

=> (a/xn)2 ≤ a | /xn2

=> a2 ≤ a/xn2 | - a2

=> 0 ≤ (a/xn2)-a2

Völliger Widerspruch, ich denke, dass mein Ansatz komplett falsch ist.

Avatar von

Tipp: \(\displaystyle x_{n+1}^2-a=\frac14\left(x_n-\frac a{x_n}\right)^2\ge0\).

Dieser Tipp hilft mir leider nicht besonders weiter. :(

Hallo Scout, bei dem zu beweisenden Sachverhalt fehlt die Initialisierung:  Was ist x0?  Wenn man das hat, könnte man das Ganze für n = 1, 2, 3 prüfen und eventuell vollständige Induktion probieren.

Ein anderes Problem?

Stell deine Frage

Willkommen bei der Mathelounge! Stell deine Frage einfach und kostenlos

x
Made by a lovely community